LSAT and Law School Admissions Forum

Get expert LSAT preparation and law school admissions advice from PowerScore Test Preparation.

 Jon Denning
PowerScore Staff
  • PowerScore Staff
  • Posts: 904
  • Joined: Apr 11, 2011
|
#37234
Question #8 is a Local question with the condition added that O is earlier than M. As a starting point this doesn't tell us much, as either M placement, 3 or 5, would still be possible and thus multiple outcomes allowed. That's also why this makes sense as a Could Be True EXCEPT question: a number of things could still be true with O ---- M.

Specifically we're asked to determine an option that CANNOT go in 5 (that's the "could except" part). To attack it, consider both options, with M 3 and M 5, thinking about that fifth position.

Clearly if M is in 5 then that's the end of the story for that option: M is in the fifth spot, so we're done. And that tells us answer choice (B) is a possibility and thus incorrect.

Now think of the other scenario where O is ahead of M: M in 3. That tells us our first four pieces! With M in 3 we have P in 1, and to get O earlier than M we have to put O into 2. Finally, we know O is two away from L, so with O in 2 we'd have L in 4:

..... P O M L (N , S , T) [NST here are uncertain, but fill the final three spaces, 5-7]

So either M is 5th, or one of N, S, and T is 5th. That rules out answers (C), (D), and (E), leaving us with answer choice (A) as correct: L can never go fifth.

Another way to approach this would be to test the answer themselves. Imagine what would happen with L in 5. First, that means M is in 3. Has to be if someone else take 5. And then O would either go 7, and thus not be ahead of M (so that's no good here), or O would go 3...but M is already there! So L in 5 leaves no way for O to be ahead of M, and again is the right answer to this question.

Get the most out of your LSAT Prep Plus subscription.

Analyze and track your performance with our Testing and Analytics Package.